1
$\begingroup$

A theorem of Duffin and Schaeffer says (quoting from R. P. Boas, Entire Functions, Theorem 10.5.1):

Let $f(z)$ be regular in $|\arg z| \leq \alpha \leq \pi / 2$ and let $h(\theta) \leq a |\cos \theta| + b |\sin \theta|$, [where $h$ is the indicator function of $f$,] and $b < \pi$. If $\{\lambda_n\}$ is an increasing sequence of real numbers such that $\lambda_{n + 1} - \lambda_n \geq 2 \delta > 0$, $|\lambda_n - n| \leq L$, $n = 1, 2, \cdots$, and $\{f(\lambda_n)\}$ is bounded, then $f(x)$ is bounded for $x > 0$; if $f(\lambda_n) \to 0$, $f(x) \to 0$ as $x \to \infty$.

Boas then says about the theorem:

The theorem is not true without some condition more restrictive than $|\lambda_n - n| = o(n)$, but it is an open question whether the condition $|\lambda_n - n| < L$ can be weakened to $|\lambda_n - n| < \epsilon(n)$ with some $\epsilon(n)$ which becomes infinite.

My questions are:

  1. Do the conclusions change if we allow $|\lambda_n - n| \to \infty$ but impose the condition that the $\lambda_n$ have larger than unit density, i.e. $\lim_{n \to \infty} n/\lambda_n = M > 1$ for some arbitrarily large $M$?
  2. Has there been progress towards concluding that $f(x)$ is bounded with a weaker condition than $|\lambda_n - n| < L$?
$\endgroup$

0

You must log in to answer this question.

Start asking to get answers

Find the answer to your question by asking.

Ask question

Explore related questions

See similar questions with these tags.